Difference between revisions of "2015 AMC 12B Problems/Problem 18"

(Problem)
(Problem)
Line 2: Line 2:
 
For every composite positive integer <math>n</math>, define <math>r(n)</math> to be the sum of the factors in the prime factorization of <math>n</math>. For example, <math>r(50) = 12</math> because the prime factorization of <math>50</math> is <math>2 \times 5^{2}</math>, and <math>2 + 5 + 5 = 12</math>. What is the range of the function <math>r</math>, <math>\{r(n): n \text{ is a composite positive integer}\}</math> ?
 
For every composite positive integer <math>n</math>, define <math>r(n)</math> to be the sum of the factors in the prime factorization of <math>n</math>. For example, <math>r(50) = 12</math> because the prime factorization of <math>50</math> is <math>2 \times 5^{2}</math>, and <math>2 + 5 + 5 = 12</math>. What is the range of the function <math>r</math>, <math>\{r(n): n \text{ is a composite positive integer}\}</math> ?
  
<math>\textbf{(A)}\; ? \qquad\textbf{(B)}\; ? \qquad\textbf{(C)}\; ? \qquad\textbf{(D)}\; ? \qquad\textbf{(E)}\; ?</math>
+
<math>\textbf{(A)}\; \text{the set of positive integers} \\
 +
\textbf{(B)}\; \text{the set of composite positive integers} \\
 +
\textbf{(C)}\; \text{the set of even positive integers} \\
 +
\textbf{(D)}\; \text{the set of integers greater than 3} \\
 +
\textbf{(E)}\; \text{the set of integers greater than 4}</math>
  
 
==Solution==
 
==Solution==

Revision as of 00:19, 4 March 2015

Problem

For every composite positive integer $n$, define $r(n)$ to be the sum of the factors in the prime factorization of $n$. For example, $r(50) = 12$ because the prime factorization of $50$ is $2 \times 5^{2}$, and $2 + 5 + 5 = 12$. What is the range of the function $r$, $\{r(n): n \text{ is a composite positive integer}\}$ ?

$\textbf{(A)}\; \text{the set of positive integers} \\ \textbf{(B)}\; \text{the set of composite positive integers} \\ \textbf{(C)}\; \text{the set of even positive integers} \\ \textbf{(D)}\; \text{the set of integers greater than 3} \\ \textbf{(E)}\; \text{the set of integers greater than 4}$

Solution

See Also

2015 AMC 12B (ProblemsAnswer KeyResources)
Preceded by
Problem 17
Followed by
Problem 19
1 2 3 4 5 6 7 8 9 10 11 12 13 14 15 16 17 18 19 20 21 22 23 24 25
All AMC 12 Problems and Solutions

The problems on this page are copyrighted by the Mathematical Association of America's American Mathematics Competitions. AMC logo.png